You are on page 1of 32

216

Data Interpretation
Directions: Read the data given in the table and graph carefully and answer the
following questions.
A watch salesman sells watches at 3 different discount percentages 10%, 20% &
25% on the marked price in five different months. Some watches were sold at
10% discount, some at 20% discount and the remaining at 25% discount.
Note: Marked price of a watch is Rs. 1000.

निर्दे श: तानिका और ग्राफ़ में नर्दए गए डे टा को ध्यािपूर्वक पढें और निम्ननिखित प्रश्ोों के


उत्तर र्दें ।

एक घड़ी नर्क्रेता पााँ च अिग-अिग मह़ीिोों में अोंनकत मूल्य पर 3 अिग-अिग छूट
प्रनतशत 10%, 20% और 25% पर घनडयााँ बेचता है । कुछ घनडयााँ 10% छूट पर, कुछ
20% छूट पर और शेष 25% छूट पर बेच़ी गईों। िोट: एक घड़ी का अोंनकत मूल्य रु. 1000.

250

200
200
160
150
150
100
Total Watches Sold
100

50

0
January February March May
Month Ratio of watches Percent of
sold at 10% watches sold at
discount to 20% 25% discount
discount
January 2:X 40%
February 10:11 30%
March --- 50%
April 3:5 60%
May --- 50%

Q1. If the total amount of discount given by the seller on the Watches in January
is Rs. 18000, then what is the value of X?

यनर्द जिर्ऱी में नर्क्रेता द्वारा घनडयोों पर र्द़ी गई छूट क़ी कुि रानश रु. 18000 है , तो X का
मूल्य क्या है ?

A. 1
B. 5
C. 3
D. 7
E. None of these

Q2. If the total amount of discount given on the watches sold at 10% discount is
Rs. 1000 less than that of sold at 20% discount in March, then what is the ratio of
watches sold at 25% discount to the watches sold at 20% discount in March?

यनर्द 10% छूट पर बेच़ी गई घनडयोों पर र्द़ी गई छूट क़ी कुि रानश रु. माचव में 20% छूट पर
बेच़ी गई घनडयोों से 1000 रु कम है , तो 25% छूट पर बेच़ी गई घनडयोों का माचव में 20%
छूट पर बेच़ी गई घनडयोों से अिुपात क्या है ?

A. 8:5
B. 8:3
C. 7:5
D. 7:4
E. None of these
Q3. If the total amount of discount given on the watches sold at 10% discount is
Rs. 1000 less than that of sold at 20% discount in March, then what is the ratio of
watches sold at 25% discount to the watches sold at 20% discount in March?

यनर्द 10% छूट पर बेच़ी गई घनडयोों पर र्द़ी गई छूट क़ी कुि रानश रु. माचव में 20% छूट पर
बेच़ी गई घनडयोों से 1000 रु कम है , तो 25% छूट पर बेच़ी गई घनडयोों का माचव में 20%
छूट पर बेच़ी गई घनडयोों से अिुपात क्या है ?

A. 100
B. 200
C. 150
D. 110
E. None of these

Q4. The difference between the total number of watches sold at 10% and 20%
discount in February is what percent less than the difference between the total
number of watches sold at 10% and 20% discount in May, if the total discount in
May is Rs. 41000?

फरर्ऱी में 10% और 20% छूट पर बेच़ी गई घनडयोों क़ी कुि सोंख्या के ब़ीच का अोंतर मई
में 10% और 20% छूट पर बेच़ी गई घनडयोों क़ी कुि सोंख्या के ब़ीच के अोंतर से नकतिा
प्रनतशत कम है , यनर्द मई में कुि छूट रु. . 41000?

A. 75%
B. 85%
C. 60%
D. 40%
E. None of these

Q5. If the total amount of discount given on the watches sold at 10% discount is
Rs. 8000 less than that sold at 20% discount in May, then what is the ratio of
watches sold at 25% discount to the watches sold at 20% discount in May?
यनर्द 10% छूट पर बेच़ी गई घनडयोों पर र्द़ी गई छूट क़ी कुि रानश रु. मई में 20% छूट पर
बेच़ी गई घनडयोों से 8000 रुपये कम है , तो 25% छूट पर बेच़ी गई घनडयोों का मई में 20%
छूट पर बेच़ी गई घनडयोों से अिुपात क्या है ?

A. 5:3
B. 4:5
C. 6:7
D. 8:5
E. None of these
Answer Key:

1. A
2. B
3. B
4. A
5. A

Solution:

Q1.
In January, watches sold at
25% Discount = 100 × 0.4 = 40,
10% Discount = x,
20% Discount = 60 – x

Total Discount
= 1000 × [40 × 0.25 + x × 0.1 + (60 – x) × 0.2] = 18000
10 + 0.1x + 12 – 0.2x = 18
0.1x = 4
x = 40
Watches sold at
10% Discount : 20% Discount = 2 : X = 40 : (60 – 40) = 40 : 20 = 2 : 1
X=1

Q2.
In March, watches sold at
25% Discount = 160 × 0.50 = 80,
10% Discount = x,
20% Discount = 80 – x
Discount on 20% – Discount on 10% = 1000
1000 × [(80 – x) × 0.2 – x × 0.1] = 1000
16 – 0.2x – 0.1x = 1
0.3x = 15
x = 50
Watch sold at,
25% Discount: 20% Discount = 80 : (80 – 50) = 80 : 30 = 8 : 3
Q3.
In April, Amount of Discount = (41000 – 31000) + 33000 = 43000 Rs.
Watches sold in April = x
1000 × x × (0.4 × 3/8 × 0.1 + 0.4 × 5/8 × 0.2 + 0.6 × 0.25) = 43000
x × (0.015 + 0.05 + 0.15) = 43
x × 0.215 = 43
x = 200

Q4.
In February, watches sold at, 20% Discount – 10% Discount
= 150 × 0.7 × (11 – 10)/21 = 5
In May, watches sold at,
25% Discount = 200 × 0.5 = 100,
10% Discount = x,
20% Discount = 100 – x
Total Discount = 1000 × [x × 0.1 + (100 – x) × 0.2 + 100 × 0.25] = 41000
0.1x + 20 – 0.2x + 25 = 41
0.1x = 4
x = 40
In May, watches sold at,
20% Discount – 10% Discount = (100 – 40) – 40 = 20
Required % = (20 – 5)/20 × 100 = 75%

Q5.
In May, Watch sold at,
25% Discount = 200 × 0.5 = 100, 10% Discount = x, 20% Discount = 100 – x
Discount on 20% – Discount on 10% = 8000
1000 × [(100 – x) × 0.2 – x × 0.1] = 8000
20 – 0.2x – 0.1x = 8
0.3x = 12
x = 40
Phone sold at, 25% Discount : 20% Discount = 100 : (100 – 40) = 100 : 60 = 5 : 3
217

Data Interpretation

Directions: Study the following information carefully and answer the related
questions.

Following line graph represents the data regarding the number of people visited
five different countries A, B, C, D and E. Each person visited to meet either Akshay
or Bharat or Sanjay.

For a particular country: M = number of adults and N = number of kids

(1) If the sum of the digits of M is an even number and sum of the digits of N is an
odd number, then the ratio of the number of people visited to meet Akshay and
Bharat is 11 : 4 respectively.

(2) If both the sum of the digits of M and the sum of the digits of N is an odd
number then the percentage break – up of the number of people visited to meet
Akshay , Bharat and Sanjay is 40%, 50% and 10% respectively.

(3) If both the sum of the digits of M and the sum of the digits of N is an even
number then the ratio of the number of people visited to meet Bharat and Sanjay
is 2 : 3 respectively.

(4) If the sum of the digits of M is an odd number and sum of the digits of N is an
even number then the percentage break – up of the number of people visited to
meet Akshay, Bharat and Sanjay is 20%, 70% and 10% respectively
निर्दे श: निम्ननिखित जािकाऱी का ध्यािपूर्वक अध्ययि करें और सोंबोंनित प्रश्ोों के उत्तर र्दें ।

निम्ननिखित रे िा ग्राफ पाों च अिग-अिग र्दे शोों A, B, C, D और E का र्दौरा करिे र्ािे िोगोों
क़ी सोंख्या से सोंबोंनित डे टा को र्दशाव ता है । प्रत्येक व्यखि या तो अक्षय या भरत या सोंजय से
नमििे गया था।

नकस़ी नर्शेष र्दे श के निए: M = र्यस्ोों क़ी सोंख्या और N = बच्ोों क़ी सोंख्या

(1) यनर्द M के अोंकोों का योग एक सम सोंख्या है और N के अोंकोों का योग एक नर्षम सोंख्या


है , तो अक्षय और भरत से नमििे आए िोगोों क़ी सोंख्या का अिुपात क्रमशः 11:4 है ।

(2) यनर्द M के अोंकोों का योग और N के अोंकोों का योग र्दोिोों एक नर्षम सोंख्या है , तो


अक्षय, भरत और सोंजय से नमििे आए िोगोों क़ी सोंख्या का प्रनतशत नर्भाजि 40%, 50%
और है । क्रमशः 10%।

(3) यनर्द M के अोंकोों का योग और N के अोंकोों का योग र्दोिोों एक सम सोंख्या है तो भरत


और सोंजय से नमििे आए िोगोों क़ी सोंख्या का अिुपात क्रमशः 2:3 है ।

(4) यनर्द M के अोंकोों का योग एक नर्षम सोंख्या है और N के अोंकोों का योग एक सम सोंख्या


है , तो अक्षय, भरत और सोंजय से नमििे आए िोगोों क़ी सोंख्या का प्रनतशत ब्रेक-अप 20%,
70 है क्रमशः % और 10%.

1200

1000 1010

850
800
800 750
650 Total
600 600
Adult
500 500
450 420 Kids
400 360 380
350
250
200
150

0
A B C D E
Q1. If the average of the number of people visited to meet Sanjay in country A, B
and C taken together is 405/3 and the respective ratio of the number of adults to
kids who visited country C to meet Akshay is 23 : 32, then approximately what
percentage of kids visited country C to meet Akshay?

यनर्द र्दे श A, B और C में सोंजय से नमििे आए िोगोों क़ी कुि सोंख्या का औसत 405/3 है
और र्दे श C में अक्षय से नमििे आए र्यस्ोों और बच्ोों क़ी सोंख्या का सोंबोंनित अिुपात
23:32 है , तो िगभग नकतिे प्रनतशत बच्े अक्षय से नमििे के निए र्दे श C गए?

A. 88.88%
B. 11.11%
C. 9.09%
D. 66.66%
E. None of these

Q2. If N is the sum between the number of people who visited country E to meet
Akshay and Bharat and N is 90 less than the sum between the number of people
who visited country A to meet Akshay and Bharat, then what is the average of
number of people who visited country A and E together to meet Sanjay?

यनर्द N, अक्षय और भरत से नमििे के निए र्दे श E में जािे र्ािे िोगोों क़ी सोंख्या के ब़ीच का
योग है और N, अक्षय और भरत से नमििे के निए र्दे श A में जािे र्ािे िोगोों क़ी सोंख्या के
ब़ीच के योग से 90 कम है , तो सोंख्या का औसत क्या है ? र्े िोग जो सोंजय से नमििे के निए
र्दे श A और E में एक साथ गए थे?

A. 20
B. 30
C. 60
D. 80
E. None of these

Q3. 300 people visited country D to meet Akshay. If 11/6 of people visited country
D to meet Akshay visited country C to meet Akshay, then what is the difference
between the number of people who visited country D and country C to meet
Bharat?
अक्षय से नमििे के निए 300 िोग कोंटर ़ी ड़ी पहों चे। यनर्द 11/6 िोग अक्षय से नमििे के निए
र्दे श D में गए, तो अक्षय से नमििे के निए र्दे श C में गए, तो भारत से नमििे के निए र्दे श D
और र्दे श C में जािे र्ािे िोगोों क़ी सोंख्या के ब़ीच क्या अोंतर है ?

A. 15
B. 10
C. 2
D. 0
E. None of these

Q4. Number of people visited country B to meet Akshay is 5/7 less than the
number of people visited same country to meet Bharat. The sum of the number of
people visited country B and country E to meet Sanjay is 135. How many people
visited country E to meet Sanjay?
अक्षय से नमििे के निए र्दे श B में जािे र्ािे िोगोों क़ी सोंख्या, भारत से नमििे के निए उस़ी
र्दे श में जािे र्ािे िोगोों क़ी सोंख्या से 5/7 कम है । सोंजय से नमििे के निए र्दे श B और र्दे श
E में जािे र्ािे िोगोों क़ी सोंख्या का योग 135 है । सोंजय से नमििे के निए नकतिे िोग र्दे श E
में गए?
A. 90
B. 50
C. 80
D. 75
E. None of these

Q5. Total number of kids is approximately what percent of total number of


adults?
बच्ोों क़ी कुि सोंख्या र्यस्ोों क़ी कुि सोंख्या का िगभग नकतिा प्रनतशत है ?
A. 61.24%
B. 59.12%
C. 45.27%
D. 75.26%
E. None of these
Answer Key:
1. A
2. D
3. D
4. D
5. B

Solution:
By Company A, Production in December = 1900 × 3 – (2000 + 1800) = 1900
Increase in sell in January = 40/3 + 20/3 = 60/3 = 20%
Unsold laptop in December = 1900 – 1200/1.2 = 1900 – 1000 = 900
Unsold laptop in December & April, by Company A = 900 + 200 = 1100
Required % = 1100/1900 × 100 = 1100/19%

Q1.
85 + 60 + (1010 – 15x) = 3 × 405/3
1155 – 15x = 405
15x = 750
x = 50
Number of kids visited Country C to meet Akshay = 11 × 50 × 32/55 = 320
% = 320/360 × 100 = 88.88%

Q2.
N = 300 + 375 = 675 = (170 + 595) – 90
Average people visited country A & E to meet Sanjay = (85 + 75)/2 = 80

Q3.
800 – 5y = 300
5y = 500
y = 100
11x = 300 × 11/6
x = 50
People visited Country C to meet Bharat – People visited Country D to meet
Bharat
= 4 × 50 – 2 × 100 = 0

Q4.
People visited Country E to meet Sanjay = 75

Q5.
Total Kids = 350 + 150 + 360 + 380 + 250 = 1490
Total Adults = 500 + 450 + 650 + 420 + 500 = 2520
% = 1490/2520 × 100 = 59.12%
218

Data Interpretation

Directions: Answer the questions based on the information given below.

Jodha, Shakir and Rana distributed Rs. [X] among themselves in the ratio 5 : 6 : 4
respectively. After spending 20% of their respective shares, Jodha and Rana
invested the remaining amount in scheme 'A' for 4 years and in scheme ―B' for 2
years, respectively. Scheme 'A' offers 20% p.a. simple interest while scheme 'B'
offers 50% p.a. compound interest, compounded annually. The interest received
by Jodha is Rs. 3200 less that of Rana. Shakir purchased a Mobile from his share of
money and sold it at 20% profit. If he had sold the Mobile at [Y]% profit, then he
would have received Rs. 1200 more. Jodha and Rana invested the interest
received by them from scheme A and scheme B, respectively, in a business
together. After 6 Months, Shakir also joined them with the sum obtained by him
on selling the Mobile. If the profit received by Shakir at the end of the year is Rs.
9000, then the total profit received by all three of them at the end of the year will
be Rs. [Z].

जोिा, शानकर और राणा िे रुपये बाों टे। [X] आपस में क्रमशः 5:6:4 के अिुपात में। अपिे
सोंबोंनित शेयरोों का 20% िचव करिे के बार्द, जोिा और राणा िे शेष रानश को क्रमशः 4
साि के निए योजिा 'ए' में और 2 साि के निए योजिा 'ब़ी' में निर्ेश नकया। स़्ीम 'ए'
20% प्रनत र्षव क़ी पेशकश करत़ी है । सािारण ब्याज जबनक योजिा 'ब़ी' 50% प्रनत र्षव
प्रर्दाि करत़ी है । चक्रर्ृखि ब्याज, र्ानषवक रूप से सोंयोनजत। जोिा को प्राप्त ब्याज रु. राणा
से 3200 कम. शानकर िे अपिे नहस्से के पैसे से एक मोबाइि िऱीर्दा और उसे 20% िाभ
पर बेच नर्दया। यनर्द उसिे मोबाइि को [Y]% िाभ पर बेचा होता, तो उसे रु. 1200 और.
जोिा और राणा िे क्रमशः स़्ीम ए और स़्ीम ब़ी से प्राप्त ब्याज को एक साथ एक
व्यर्साय में निर्ेश नकया। 6 मह़ीिे के बार्द, शानकर भ़ी मोबाइि बेचिे पर प्राप्त रानश से
उिके साथ जुड गया। यनर्द र्षव के अोंत में शानकर को प्राप्त िाभ रु. 9000 है ,
तो र्षव के अोंत में उि त़ीिोों द्वारा प्राप्त कुि िाभ रु. [Z] होगा।
Q1. 'Z' is how much percent more/less than the amount received by Shakir after
selling the Mobile at 20% profit?

'Z' शानकर द्वारा 20% िाभ पर मोबाइि बेचिे के बार्द प्राप्त रानश से नकतिा प्रनतशत
अनिक/कम है ?

A. 4.3% more
B. 4.5% less
C. 6.25% less
D. 6.9% less
E. None of these

Q2. Find the difference between values of X and Z.

X और Z के मािोों के ब़ीच अोंतर ज्ञात क़ीनजए।

A. 25000
B. 10000
C. 11000
D. 33000
E. None of these

Q3. If Shakir had invested 75% of the share received by him out of Rs. X at 20%
p.a. for 3 years, then find the simple Interest he would have received.

यनर्द शानकर िे रुपये में से प्राप्त शेयर का 75% निर्ेश नकया होता। X 20% प्रनत र्षव 3 र्षों
के निए, तो उसे प्राप्त होिे र्ािा सािारण ब्याज ज्ञात क़ीनजए।

A. 10800
B. 10000
C. 11000
D. 26000
E. None of these
Q4. Find the value of Z : (2Z – 36).

Z का माि ज्ञात क़ीनजये: (2Z – 36).

A. 15:28
B. 51:23
C. 51:25
D. 25:14
E. None of these

Answer key:

1. C
2. C
3. A
4. A

Solutions:

Share of, Jodha = 5x Rs. , Shakir = 6x Rs. , Rana = 4x Rs.


4x × 0.8 × (1.52 – 1) – 5x × 0.8 × 0.2 × 4 = 3200
3.2x × 1.25 – 3.2x = 3200
0.25x = 1000
x = 4000 Rs.
X = (5 + 6 + 4) × 4000 = 15 × 4000 = 60000 Rs.
6 × 2000 × (1 + Y/100) = 6 × 2000 × 1.2 + 1200
10 × (1 + Y/100) = 10 × 1.2 + 1
10 + Y/10 = 12 + 1
Y/10 = 13 – 10
Y = 3 × 10 = 30
Interest received by Jodha = 3.2 × 4000 × 1.25 = 16000 Rs.
Interest received by Rana = 3.2 × 4000 = 12800 Rs.
Sum obtained by Shakir = 6 × 4000 × 1.2 = 28800 Rs.
Profit Ratio, A : B : C = (160 × 12) : (128 × 12) : (288 × 6) = 10 : 8 : 9
Total Profit = Z = 9000 × 27/9 = 27000 Rs.
Q1.

Required % = (28800 – 27000)/28800 × 100 = 6.25% Less

Q2.

X – Z = 60000 – 27000 = 33000

Q3.

SI received by Shakir = 6 × 4000 × 0.75 × 0.2 × 3 = 10800 Rs.

Q4.

Z : (2Z – 36) = 27000 : (2 × 27000 – 3600) = 27000 : 50400 = 15 : 28


219

Miscellaneous:
Q1. Which of the following is true about the relation between the wrong numbers
in the following series?
I: 7, 14, 43, 104, 233, 446
II: 39, 81, 138, 209, 299, 409

a. The difference between the wrong term of I and II when divided by 4 leaves
the remainder 0.
b. 3/2 of the sum of the wrong numbers is divisible by 13.
c. 19 is added to the wrong number in I, then divided by 3 so the difference
between the number obtained and wrong number of II is an odd number.

A. Only a
B. Only a & c
C. Only b
D. Only a & b
E. None of a, b & c

Q2. I. 25(𝑥 − 1) − 55(𝑥 + 2) = 45


II. 20𝑦 4 = (−12 ∗ 4)𝑦 2 − 76𝑦 3
A. x>y
B. x<y
C. x≥y
D. x≤y
E. x=y or no relation can be established

Q3.
I. 3𝑥 + 4𝑦 = 880
II. 9𝑥 − 8𝑦 = 240

A. x>y
B. x<y
C. x≥y
D. x≤y
E. x=y or no relation can be established

Q4. Solve the question given below based on the series given below.
51, 55, 64, 80, 105 , __
700 is the nth term of this series.
Find ‘n’.

A. 8th
B. 9th
C. 12th
D. 11th
E. Cannot be determined

Q5. Solve the question given below based on the series given below.
4, 21, (Q), 61, 84, 109 , __
264 is the nth term of this series.

A. 8th
B. 9th
C. 12th
D. 11th
E. Cannot be determined
Answer Key:

1. B
2. B
3. B
4. C
5. D

Solution:
Q1 ->
I: 14 is the wrong term
23 – 3 = 5 (12-7)
33 + 4 = 31(43-12)
43 -3 = 61 (104-43)
53 +4 =129 (233-104)
63 -3 = 213 (446-233)
II: 138 is the wrong term
+(𝟔𝟐+𝟔),+(𝟕𝟐+𝟕),+(𝟖𝟐+𝟖),+(𝟗𝟐+𝟗),+(𝟏𝟎𝟐+𝟏𝟎)

Now, a is correct as 4*31 = 124, b is incorrect as on dividing by 13 the remainder


is not 0, c is correct as the remainder is 127 which is odd.

Q2. ->
X= -6
Y= 3, 4/5
So, 𝑥 < 𝑦

Q3 ->
X= 666.66
Y= 720
So, 𝑥 < 𝑦
Q4. Pattern is of double difference,
51, 55, 64, 80, 105
55-51 = 4 (22)
64-55 = 9 (32)
80-64 = 16
Similarly 700 is the 12th term

Q5 ->
Pattern is of double difference,
4, 21, (Q), 61, 84, 109
21-4 = 17
40-21 = 19
61-40 = 21
Here numbers are increasing in odd number differences.
Thus 264 is the 11th term
220

Statement Based Questions:


Q1. The question given below consists of a question and 3 statements given
below it. You must decide whether the data provided in the statements is
sufficient to answer the question.
Train P takes 9 seconds to cross a pole and 20 seconds to cross platform X. Find
the time taken by Train P to cross a platform of length 240 meters.
Statement I: The length of Train P and Platform X are in ratio 9:11.
Statement II: Difference between the lengths of train P and Platform X is 60
meters.
Statement III: Train P takes 22 seconds to cross a bridge that is 60 meters longer
than platform X.

ि़ीचे नर्दए गए प्रश् में एक प्रश् और उसके ि़ीचे 3 कथि नर्दए गए हैं । आपको यह तय करिा
होगा नक कथिोों में नर्दया गया डे टा प्रश् का उत्तर र्दे िे के निए पयाव प्त है या िह़ीों।

टर े ि P को एक िोंभा पार करिे में 9 सेकोंड और प्लेटफ़़ॉमव X को पार करिे में 20 सेकोंड
िगते हैं । 240 म़ीटर िोंबे प्लेटफ़़ॉमव को पार करिे में टर े ि P द्वारा निया गया समय ज्ञात
क़ीनजए।

कथि I: टर े ि P और प्लेटफ़़ॉमव X क़ी िोंबाई 9:11 के अिुपात में है।

कथि II: टर े ि P और प्लेटफ़़ॉमव X क़ी िोंबाई के ब़ीच का अोंतर 60 म़ीटर है ।

कथि III: टर े ि P को प्लेटफ़़ॉमव X से 60 म़ीटर िोंबे पुि को पार करिे में 22 सेकोंड का समय
िगता है ।
A. Only I and II together are sufficient.
B. III is alone sufficient,
C. II alone is sufficient.
D. Neither I, II nor III together are sufficient.
E. Either II alone or III alone is sufficient.

Q2. Difference between the curved surface area of cone and cylinder is 64 pie (I.e.
64*22/7) and height of cylinder is __ % more than its radius. If radius of cylinder is
__ % more than radius of cone, then the volume of cylinder will be 864 pie (i.e.
864*22/7).

शोंकु और बेिि के र्क्र पृष्ठ क्षेत्रफि के ब़ीच का अोंतर 64 पाई (अथाव त 64*22/7) है और
बेिि क़ी ऊोंचाई उसक़ी नत्रज्या से __% अनिक है । यनर्द बेिि क़ी नत्रज्या शोंकु क़ी नत्रज्या से
__% अनिक है , तो बेिि का आयति 864 पाई (अथाव त 864*22/7) होगा।

A. 30 and 20
B. 15 and 40
C. 20 and 60
D. 25 and 50
E. None of these

Q3-4) Punit, Pratham and Purvi started a business with initial investment of
Rs.10000, Rs.15000 and Rs. 16000 respectively for ‘m+4’ years, ‘m’ years and ‘m-
2’ years respectively. During the period for which Pratham kept his investments
he also handled the accounts of the business and thus received a salary of
Rs.2000 each year from the gross profit of the business. At the end of business,
the amount received by Pratham is Rs.21000 more than that received by Purvi
and company received a gross profit of Rs.86000.

पुनित, प्रथम और पूर्ी िे 10000 रुपये, 15000 रुपये और 15000 रुपये के शुरुआत़ी
निर्ेश के साथ एक व्यर्साय शुरू नकया। क्रमशः 'm+4' र्षव, 'm' र्षव और 'm-2' र्षव के
निए क्रमशः 16000। नजस अर्नि के निए प्रथम िे अपिा निर्ेश रिा, उस अर्नि के
र्दौराि उन्ोोंिे व्यर्साय के िातोों को भ़ी सोंभािा और इस प्रकार व्यर्साय के सकि िाभ से
प्रत्येक र्षव 2000 रुपये का र्ेति प्राप्त नकया। व्यर्साय के अोंत में , प्रथम को प्राप्त रानश
पूर्ी को प्राप्त रानश से 21000 रुपये अनिक है और कोंपि़ी को 86000 रुपये का सकि
िाभ प्राप्त हआ।

Q3. What is the profit received by Punit?

पुनित को नकतिा िाभ प्राप्त हआ?

A. Rs.21000
B. Rs.24000
C. Rs.25000
D. Rs.27000
E. None of these

Q4. What is difference between the average of total earnings of Punit and
Pratham together and average of total earnings of Pratham and Purvi together?

पुनित और प्रथम क़ी कुि कमाई के औसत और प्रथम और पूर्ी क़ी कुि कमाई के औसत
के ब़ीच नकतिा अोंतर है ?

A. Rs.5000
B. Rs.2500
C. Rs.2000
D. Rs.1500
E. None of these
Q5. How many students from Section P got selected?

I. Number of students studying in Section P and Section Q are in the ratio of 2 : 3


respectively and 50% of the students studying in Section Q got selected.

II. Number of students who got selected from Section Q is 125% of the number of
students who got selected from Section P.

सेक्शि प़ी से नकतिे छात्रोों का चयि हआ?


I. सेक्शि P और सेक्शि Q में पढिे र्ािे छात्रोों क़ी सोंख्या क्रमशः 2:3 के अिुपात
में है और सेक्शि Q में पढिे र्ािे 50% छात्रोों का चयि नकया गया है।
II. अिुभाग Q से चयनित छात्रोों क़ी सोंख्या अिुभाग P से चयनित छात्रोों क़ी सोंख्या का
125% है ।
A. Only I is sufficient
B. I and II together are sufficient
C. Only II is sufficient
D. Both statements together are not sufficient
E. None of these
Answer Key:

1. E
2. C
3. C
4. B
5. E

Solution:
Q1 ->
Before proceeding with the question we should know that, since length of a pole
is negligible, the time taken to cross a pole is considered time taken by a train to
cross its own length.
From the passage we can state that the Train P covers its own length in 9 seconds
and Train P covers its own length +length of platform X in 20 seconds. So it covers
length of Platform X in (20-9) = 11 seconds.
Thus we can say ratio of speeds = ratio of length (as speed is constant)
So length of train P: length of platform X = 9:11
Statement I – the data in this statement is already given in the question.
Hence Statement I is not sufficient. It’s a Redundant Statement.
Statement II = here the difference of lengths is given and we know from the
question the ratio of lengths so, we have
11u – 9u = 60
1u = 30
So length of platform X= 11*30 = 330m & length of train P = 270 m
So speed is = (330+270)/20 = 30m/s
Time to cross platform of length 240m is = (270+240)/30 = 17 seconds
So Statement II is alone sufficient.
Statement III –
We know Train P covers its own length + length of Platform P in 20 sec
And Train P covers its own length + length of bridge in 22 sec
So to cover the extra distance i.e. Length of Bridge – Length of Platform x, Train p
takes 2 seconds at a constant speed.
So 60 m is covered in 2 seconds, thus Speed of Train P is (60/2) = 30 m/s.
Length of train P = speed * time taken to cross a pole = 30*9 = 270 m
Time to cross platform of length 240m is = (270+240)/30 = 17 seconds
So Statement III is alone sufficient.
Thus Statement II alone And Statement III alone are sufficient.

Q2->Let slant height of cone be L, radius of cone be R and radius of cylinder be r


and height of cylinder be h
Given,
22/7*(R)*L – 2*22/7* (r)*h = 64 *22/7 ---- (Equation I)
[Now we know L has to be a natural number as the equation above contains no
root, as L2 = R2 + h2.]
From Option I
R = 10u and L = 13u and r = 12u
Method 1:
L: R = 13:10
So h2 = 132 – 102 = 69 – which is not a perfect square.
So option I is not correct.
Method 2: (putting values in Equation I)
22/7*10u*13u – 2*22/7 * 12u * [(13u)2 –(10u)2]1/2 = 64*22/7
Here again we won’t get a numerical value of h as a natural number.
So I is not correct.

Similarly checking option B and D are also not correct,


But from option c we have
L = 5u, R = 4u, so r = 6u
h = [25-16] ½ = 91/2 = 3u
Thus,
22/7*4u*5u – 2*22/7 * 6u *3u = 64*22/7
22/7 [20u2-36u2] =64*22/7
16u2= 64
U2 = 4
U=2
So, Volume of cylinder = 22/7*r2 * h = 22/7 *(12)2 * 6 = 22/7*144*6 = 22/7*864
Thus option C is correct.

Q3-4) ->
Punit Pratham Purvi
Investment 10 : 15 : 16
Time (m+4) : (m) : (m-2)
Profit Ratio 10m+40 : 15m : 16m-32
And Pratham also receives 2000 each year, so he gets (2000m) in addition to his
profit share.
Now with the given share of Pratham and Gross Profit we could determine the
value of m through an equation , but that would be a lengthy process, instead we
check some values of m, where m>2 (as Purvi invested for m-2 years and time
cannot be negative or 0).
If m =3, then
Ratio of time will be – Punit: Pratham: Purvi = 7:3:1
Profit ratio= Punit: Pratham: Purvi = (10*7): (15*3): (16*1) = (70:45:16)
And salary of Pratham as accountant will be = 2000*3 = Rs.6000
Now, Net Profit = 86000-6000 = Rs. 80000
And Profit share of Pratham should be
80000*45/131 +6000 = 27480.91 +6000 = 33840.91
And Profit share of Purvi should be
80000*16/131 =9770.99
Difference is not equal to Rs.21000
After checking for m=4, 5, 6, 7 and 8
We get from m= 8
Profit Ratio = 5:5:4
And net Profit = 70000
Checking we get,
Profit share of Pratham should be = 70000*5/14 + 16000 = 41000
Profit share of Purvi should be = 70000*4/14 = 20000
Difference = 21000 Rs.
Q3 -> Profit share of Punit is = 70000*5/14 = 25000Rs.
Q4 -> Required % = [(25000+41000) – (41000+20000)]/2
=5000/2 = 2500 Rs.
Q5. From (I), Upstream Speed = x km/h, Downstream Speed = y km/h
460/x + 460/y = 46
1/x + 1/y = 1/10
From (II), y = 50, x = (50 × 10)/(50 – 10) = 12.5 km/h
From (III), x = 25/2 = 12.5, y = (12.5 × 10)/(12.5 – 10) = 50 km/h
Speed of Stream = (50 – 12.5)/2 = 18.75 km/h
Either I & II or I & III are sufficient

You might also like